Integrating Int((5+10y^4)dy/(y+2y^5)) - A Step-by-Step Guide

  • Thread starter Thread starter marmot
  • Start date Start date
  • Tags Tags
    Integrating
Click For Summary
The integral of (5 + 10y^4) dy / (y + 2y^5) can be simplified through substitution, specifically using u = y + 2y^5, which leads to a cancellation with the numerator. The numerator's structure suggests factoring, which can simplify the integration process. A suggested approach involves recognizing that 5 + 10y^4 can be expressed as 4 + 1 + 10y^4, allowing for a crafty substitution. After factoring and canceling common terms, the integration becomes straightforward. The discussion emphasizes the importance of substitution and factoring in solving the integral effectively.
marmot
Messages
55
Reaction score
1

Homework Statement



I want to deal with this int((5+10y^4)dy/(y+2y^5))

Homework Equations



integration, substitution, partial fractions?


The Attempt at a Solution



I tried a bunch of random things. I think it hs to do with substitution because if I make u=y+2y^5, du/dy=1+10y^4 which is strikingly similar to the numerator, so there must be a cancellation. this integral is part of a dif equation, but i can't see to go past this! i know for sure this is the correct set up because i used my calculator to integrate and it solved the differential equation.
 
Physics news on Phys.org
Uh, the ratio of those two polynomials is a VERY SIMPLE THING. Can you find it? Factor them.
 
Try this :
5 + 10y^4 = 4 + 1 + 10y^4

after you get it, think of a crafty substitution
 
aostraff said:
Try this :
5 + 10y^4 = 4 + 1 + 10y^4

after you get it, think of a crafty substitution

Think of a crafty cancellation before you do the crafty substitution.
 
jesus i feel like a complete retard. i hate when i don't get something as obvious.i factored both and canceled that nasty factor and then everything went smooth, thanks a bunch gentlemen.
 
Question: A clock's minute hand has length 4 and its hour hand has length 3. What is the distance between the tips at the moment when it is increasing most rapidly?(Putnam Exam Question) Answer: Making assumption that both the hands moves at constant angular velocities, the answer is ## \sqrt{7} .## But don't you think this assumption is somewhat doubtful and wrong?

Similar threads

Replies
20
Views
2K
Replies
3
Views
2K
Replies
2
Views
1K
  • · Replies 6 ·
Replies
6
Views
1K
Replies
11
Views
2K
Replies
5
Views
2K
  • · Replies 3 ·
Replies
3
Views
2K
Replies
7
Views
1K
Replies
2
Views
1K
  • · Replies 12 ·
Replies
12
Views
3K